Jump to content

toanND's Content

There have been 50 items by toanND (Search limited from 05-06-2020)



Sort by                Order  

#722758 Hướng vẽ hình bằng Geogebra

Posted by toanND on 05-06-2019 - 10:11 in Thử các chức năng của diễn đàn

[attachment=Capture 6.png]




#722132 Chứng minh rằng số đo góc MPN luôn không đổi khi D thay đổi

Posted by toanND on 11-05-2019 - 12:27 in Hình học

Cho tam giác ABC, giả sử có điểm P nằm trong tam giác ABC sao cho góc BPC = góc CPA = góc APB. PB, PC theo thứ tự cắt CA, AB tại E, F. D là điểm di chuyển trên cạnh BC. Đường thẳng DF cắt đường thẳng AC tại M. Đường thẳng DE cắt đường thẳng AB tại N.

1. Chứng minh rằng số đo góc MPN luôn không đổi khi D thay đổi.

2. Gọi giao điểm của đường thẳng EF với đường thẳng MN là Q. Chứng minh rằng PQ là phân giác của góc MPN.

Capture 4.PNG




#722159 Chứng minh rằng số đo góc MPN luôn không đổi khi D thay đổi

Posted by toanND on 12-05-2019 - 17:01 in Hình học

Xét phép nghịch đảo tâm $P$ phương tích bất kì, ta đưa bài toán đã cho về bài toán mới sau.

Bài toán mới. Cho $\Delta ABC,P$ là điểm trong tam giác thoả $\widehat{APB}= \widehat{BPC}= \widehat{CPA}.PB,PC$ theo thứ tự cắt $(CPA),(APB)$ tại $E,F.D$ là điểm di chuyển trên $(PBC),(PDF)$ cắt $(PAC)$ tại $M,(PDE)$ cắt $(PAB)$ tại $N.$

a) Chứng minh số đo góc $\widehat{MPN}$ không đổi.

b) $(PEF)$ cắt $(PMN)$ tại $Q.$ Chứng minh $PQ$ là phân giác $\widehat{MPN}.$

Chứng minh.

a) Gọi $G,H,I,J,K,L,O$ là tâm các đường tròn $(PAC),(PAB),(PDF),(PDE),(PBC),(PEF),(PMN).$

Từ cách xác định các điểm này, ta có các bộ điểm thẳng hàng: $\overline{L,H,I}, \overline{I,K,J}, \overline{J,G,L}, \overline{I,O,G}, \overline{J,O,H}.$

Ta cũng có $IL \perp PC,JL \perp PB,GK \perp PC,HK \perp PB,HG \perp PA.$ Với chú ý $\widehat{APB}= \widehat{BPC}= \widehat{CPA}=120^0,$ ta được $\Delta KGH$ đều và $\widehat{KHI}= \widehat{KGI}=60^0.$

Do đó $IH \parallel GK,HK \parallel GJ \Rightarrow \Delta HIK \sim \Delta GKJ \Rightarrow \frac{HI}{HG}= \frac{HI}{HK}= \frac{GK}{GJ}= \frac{GH}{GJ} \Rightarrow \Delta IHG \sim \Delta HGJ$

$\Rightarrow \widehat{IOH}= \widehat{OGH}+ \widehat{OHG}= \widehat{OHG}+ \widehat{OJG}= 180^0- \widehat{HGJ}=60^0.$

Mà $MP \perp OI,NP \perp OH \Rightarrow \widehat{MPN}=60^0.$ Ta có đpcm.

b) Theo cách xác định các tâm đường tròn ở câu a), $\Delta HLG$ đều. Lại có $\widehat{IOH}=60^0$ theo câu a) nên $L$ là trung điểm cung $HG$ không chứa $O$ của $(HOG) \Rightarrow OL$ là phân giác $\widehat{HOG}.$

Lại có $MP \perp OG,NP \perp OH,PQ \perp OL \Rightarrow PQ$ là phân giác $\widehat{MPN}.$ Ta có đpcm.

 

có cách nào k sử dụng phép nghịch đảo k ạ? Tại e chưa học cái này




#722762 CM: B,O,D thẳng hàng

Posted by toanND on 05-06-2019 - 10:28 in Hình học

Từ câu b ta có $\frac{AB}{BH}=\frac{EK}{OK}\Rightarrow \frac{AB}{2BH}=\frac{EK}{2OK}\Leftrightarrow \frac{AB}{BC}=\frac{EK}{AK}$

Xét hai tam giác ABC và EKA có $\widehat{ABC}=\widehat{AKE}; \frac{AB}{BC}=\frac{EK}{AK}$

 nên hai tam giác đó đồng dạng theo trường hợp c.g.c

suy ra $\widehat{EAK}=\widehat{ACB}=\widehat{AKB}\Rightarrow AE|| BK\Rightarrow AE\perp AB$.

Từ đó dễ dàng suy ra đpcm

Capture 6.PNG




#723048 bất đẳng thức

Posted by toanND on 14-06-2019 - 16:37 in Bất đẳng thức và cực trị

Ngưỡng mộ anh toanND quá, anh chỉ cho em cách học giỏi bđt với ạ

Kiếm sách với tài liệu mà đọc thôi e :ukliam2:




#723109 bất đẳng thức

Posted by toanND on 17-06-2019 - 12:51 in Bất đẳng thức và cực trị

Áp dụng BĐT $b^2+bc+c^2\geq\frac{3}{4}(b+c)^2$

Ta có $15\geq3a^2+4(b^2+bc+c^2)\geq3[a^2+(b+c)^2]\geq\frac{3}{2}(a+b+c)^2$

$\Rightarrow (a+b+c)^2\leq10\Rightarrow-\sqrt{10}\leq a+b+c\leq \sqrt{10}$

Từ đó ta có min , max

e tự tìm dấu = nhé




#723098 bất đẳng thức

Posted by toanND on 16-06-2019 - 22:23 in Bất đẳng thức và cực trị

BÀI 3. Áp dụng BĐT Bunhiacopxki ta có $(a^2+b)(1+\frac{1}{b})\geq(a+1)^2$

Tương tự với các BĐT còn lại rồi nhân lại ta có $(a^2+b)(b^2+c)(c^2+a)\frac{(a+1)(b+1)(c+1)}{abc}\geq(a+1)^2(b+1)^2(c+1)^2$

$\Leftrightarrow (a^2+b)(b^2+c)(c^2+a)\geq abc(a+1)(b+1)(c+1)$

Dấu = xảy ra khi a =b =c 




#723026 bất đẳng thức

Posted by toanND on 14-06-2019 - 08:50 in Bất đẳng thức và cực trị

BÀI 1

a. Áp dụng BĐT Holder ta có: $P=(\frac{1}{a}+1)(\frac{1}{b}+1)(\frac{1}{c}+1)(\frac{1}{d}+1)\geq (\sqrt[4]{\frac{1}{abcd}}+1)^{4}$

Mặt khác theo BĐT AM-GM: $\sqrt[4]{abcd}\leq \frac{a+b+c+d}{4}=\frac{1}{4}$

$\Rightarrow P\geq(4+1)^{4}=625$

Vậy $minP=625$ khi $a=b=c=d= \frac{1}{4}$

b. Ta có $Q=\frac{a(b+c+d)}{\frac{a^{2}}{3}+b^{2}+\frac{a^{2}}{3}+c^{2}+\frac{a^{2}}{3}+d^{2}}\leq\frac{a(b+c+d)}{\frac{2}{\sqrt{3}}(ab+ac+ad)}=\frac{\sqrt{3}}{2}$

Vậy $maxQ = \frac{\sqrt{3}}{2}$ khi ..........

BÀI 2. Ý tưởng cũng giống bài 1b thôi e  :closedeyes:




#723037 bất đẳng thức

Posted by toanND on 14-06-2019 - 13:22 in Bất đẳng thức và cực trị

Em cảm ơn anh toannd, bài 1 câu a anh có thể giải bằng cauchy dc k ah

Có thể dùng AM - GM (Cauchy) kiểu này : $\frac{1}{a}+\frac{1}{b}\geq\frac{4}{a+b}; \frac{1}{ab}\geq\frac{4}{(a+b)^2}$

Áp dụng hai BĐT trên, ta biến đổi biểu thức P như sau:

$P=(1+\frac{1}{a}+\frac{1}{b}+\frac{1}{ab})(1+\frac{1}{c}+\frac{1}{d}+\frac{1}{cd})\geq[1+\frac{4}{a+b}+\frac{4}{(a+b)^2}][1+\frac{4}{c+d}+\frac{4}{(c+d)^2}]=[(\frac{2}{a+b}+1)(\frac{2}{c+d}+1)] ^2 ]$

Đặt $A=[(\frac{2}{a+b}+1)(\frac{2}{c+d}+1)]^2$

$\Rightarrow A=[\frac{4}{(a+b)(c+d)}+\frac{2}{a+b}+\frac{2}{c+d}+1]^2\geq[\frac{16}{(a+b+c+d)^2}+\frac{8}{a+b+c+d}+1]^2=625$

$\Rightarrow P\geq A\geq625$  ~O)




#724550 CMR $ TM // BC $

Posted by toanND on 08-08-2019 - 09:38 in Hình học

Bài này có thể giải bằng định lý Pascal




#723405 Hê thức lượng

Posted by toanND on 29-06-2019 - 22:33 in Hình học

e coi lại đề bài 2 xem 

nhimtom



#723413 Hê thức lượng

Posted by toanND on 30-06-2019 - 16:14 in Hình học

Em cảm ơn anh toanND, em check lại bài 2 đề bài vẫn đúng ah

Ở câu b/ nếu $\angle BCA=90^0$ thì tam giác ABC có hai góc vuông à?




#723414 Hê thức lượng

Posted by toanND on 30-06-2019 - 16:49 in Hình học

$\boxed{4}$

hethuc2.PNG

a. Ta có $BK^2=BA^2=BH.BC\Rightarrow \bigtriangleup BKH \sim \bigtriangleup BCK\Rightarrow \widehat{BKH}=\widehat{BCK}$

   Tương tự $\bigtriangleup DKL \sim \bigtriangleup DCK\Rightarrow \widehat{DKL}=\widehat{BCK}$

   Vậy $\widehat{BKH}=\widehat{DKL}$

b. Dựng hai đường tròn $(B,BK)$  $(D,DK)$. Gọi T là giao điểm thứ hai của hai đường tròn này.

    Gọi M, N lần lượt là giao điểm của TK với AC, BC.

    Dễ thấy AC là tiếp tuyến chung của (B,BK) và (D, DK)

    Ta có $MA^2=MK.MT=MI^2\Rightarrow MA=MI$. Lại có $MN\parallel AH\parallel IL$ (cùng vuông BC) nên NH = NL.

    Mặt khác $KN\perp HL$ nên tam giác HKL cân $\Rightarrow KH=KL$

c. Từ $\bigtriangleup DKL \sim\bigtriangleup DCK$ kết hợp với KH = KL ta có $\widehat{KHC}=\widehat{KLH}=\widehat{DKC}$

    Do đó $\bigtriangleup CKD\sim\bigtriangleup CHK\Rightarrow CK^2=CD.CH$. Lại có tứ giác AIDH nội tiếp nên $CD.CH=CI.CA$

    Vậy $CK^2=CA.CI$  ~O)

 




#723404 Hê thức lượng

Posted by toanND on 29-06-2019 - 22:28 in Hình học

$\boxed{3}$

Kẻ EK, FI lần lượt vuông góc với AB, AC tại K, I.

Ta có $\frac{AK}{BK}=\frac{AK.AB}{BK.AB}=\frac{EA^2}{EB^2}=4=\frac{HC}{HB}$

theo định lý Thales đảo $\Rightarrow HK||AC$. Tương tự $HI\parallel AB$

$\Rightarrow AIHK$ là hình bình hành

Ta có $\bigtriangleup AEB\sim \bigtriangleup CFA \Rightarrow \measuredangle BAE=\measuredangle ACF=\measuredangle AFI\Rightarrow \bigtriangleup AEK\sim \bigtriangleup FAI$

$\Rightarrow \frac{EK}{AK}=\frac{AI}{FI}\Rightarrow \frac{EK}{AI}=\frac{AK}{FI}$. Mà $AK = HI,AI=HK$

$\Rightarrow \frac{EK}{HK}=\frac{HI}{IF}$. Lại có $\measuredangle EKH=\measuredangle BKH +90^0=\measuredangle HIC+90^0=\measuredangle HIF$

Do đó $\bigtriangleup EKH\sim\bigtriangleup HIF\Rightarrow\measuredangle HEK=\measuredangle IHF$

Vậy $\measuredangle EHF=\measuredangle EHK+\measuredangle KHI+\measuredangle IHF=(\measuredangle EHK+\measuredangle HEK)+\measuredangle BKH=180^0-\measuredangle EKH+\measuredangle BKH=180^0-90^0=90^0$

 




#723400 Hê thức lượng

Posted by toanND on 29-06-2019 - 17:40 in Hình học

$\boxed{\text{1}}$

hethuc1.PNG

a. Dễ thấy tứ giác KIHB nội tiếp $\rightarrow CI.CK=CH.CB=CA^2$

b. theo tính chất đối xứng thì $\measuredangle BDC=\measuredangle BAC=90^0;AC=DC$

Vì $\measuredangle BKC=\measuredangle BAC=\measuredangle BDC=90^0$ nên $B,K,A,C,D$ cùng thuộc một đường tròn

$\Rightarrow \measuredangle CKD=\measuredangle DAC=\measuredangle ADC=\measuredangle AKC$

ta có đpcm

 




#724490 $\widehat{NLP}=90^0$

Posted by toanND on 05-08-2019 - 21:44 in Hình học

capture NVL.PNG

Gọi AQ là đường đối trung ứng với đỉnh A của tam giác ABC. AQ cắt (O) lần thứ hai tại T

$\Rightarrow$ Tứ giác ABTC điều hòa $\Rightarrow$ SA tiếp xúc với (O) tại A (S là giao của OL với BC).

Gọi $H_{a}$ là chân đường cao kẻ từ A của tam giác ABC. K là giao của OL với $AH_{a}$.

$\Rightarrow$ K là trực tâm tam giác AQS $\Rightarrow QK \perp AS$

Gọi M, I lần lượt là trung điểm của BC, $H_{b}H_{c}$. Khi đó M, I, N thẳng hàng và $MN\perp H_{b}H_{c}$

Mặt khác dễ thấy $AS \parallel H_bH_c$ $\Rightarrow QK \parallel MN$

$KLQH_a$ nội tiếp $\Rightarrow \widehat{H_aLQ}=\widehat{H_aKQ}=\widehat{H_aNI}(KQ\parallel MN)$

$\Rightarrow NILH_a$ nội tiếp. Mặt khác $I,H_a$ cùng nằm trên đường tròn đường kính NP

Suy ra L nằm trên đường tròn đường kính NP$\Rightarrow \widehat{NLP}=90^0$ (đpcm)  :like




#720532 $2(x^{2}-4x+6)-3\sqrt[3]{x^{3}+27}=0...

Posted by toanND on 28-02-2019 - 20:49 in Phương trình - Hệ phương trình - Bất phương trình

Giải phương trình sau : $2(x^{2}-4x+6)-3\sqrt[3]{x^{3}+27}=0$




#723698 Một số bài toàn bất đẳng thức mình cần được giúp đỡ!

Posted by toanND on 12-07-2019 - 16:52 in Bất đẳng thức và cực trị

$\boxed{16}$

Áp dụng BĐT Cauchy Schwarz ta có

$\frac{a}{4b^2+1}+\frac{b}{4c^2+1}+\frac{c}{4a^2+1}=\frac{a^3}{4a^2b^2+a^2}+\frac{b^3}{4b^2c^2+c^2}+\frac{c^3}{4c^2a^2+c^2}\geq \frac{(a\sqrt{a}+b\sqrt{b}+c\sqrt{c})^2}{4(a^2b^2+b^2c^2+c^2a^2)+a^2+b^2+c^2}$

Ta chỉ cần chứng minh 

$4(a^2b^2+b^2c^2+c^2a^2)+a^2+b^2+c^2\leq 1 =(a+b+c)^2$

$\Leftrightarrow ab(1-2ab)+bc(1-2bc)+ca(1-2ca)\geq0$

Lại có $ab>0$ , $ab\leq\frac{(a+b)^2}{4}<\frac{(a+b+c)^2}{4}$ $\Rightarrow ab<\frac{1}{4}\Rightarrow ab(1-2ab)>0$

Tương tự ta có đpcm

Dấu = không xảy ra




#722559 Dựng đường tròn $\omega _{1}$

Posted by toanND on 27-05-2019 - 21:13 in Hình học

Cho đường tròn (O), dây cung AB bất kì và một điểm D nằm trên (O) khác A và B.

a) Dựng đường tròn $\omega _{1}$ và $\omega _{2}$ lần lượt tiếp xúc trong, tiếp xúc ngoài với (O) và tiếp xúc AB.

b) Kẻ dây cung DE cắt đoạn AB tại C. Dựng đường tròn $\omega _{3}$ tiếp xúc với CD, CA và tiếp xúc trong với (O). 




#722771 Dựng đường tròn $\omega _{1}$

Posted by toanND on 05-06-2019 - 17:22 in Hình học

 

b) Cách dựng $\omega_{3}:$ Gọi tâm nội tiếp $\Delta DAE$ là $L,O_3$ là hình chiếu $L$ lên phân giác trong $\widehat{ACD}.$ Khi đó đường tròn tâm $O_3$ tiếp xúc $CA$ chính là $\omega_{3}$ (bổ đề Sawayama-Thebault).

sao k đúng nhỉ?

@halloffame: Mình chỉnh lại rồi nhé.




#722765 Chứng minh BK chia đôi MF.

Posted by toanND on 05-06-2019 - 11:11 in Hình học

Capture 7.PNG

dễ thấy E là điểm chính giữa cung lớn BC nên E, O, M thẳng hàng và $EM\perp BC$.

Gọi I là giao điểm của FM với AC. Dễ thấy FM là đường thẳng Simson của tam giác ABC nên $EI \perp AC$.

Lại có $\widehat{FAE}=\widehat{FEI}\Rightarrow \Delta AEF=\Delta AEI\Rightarrow AF=AI,EF=EI$

do đó AE là trung trực của FI $\Rightarrow AK\perp FM$. (1)

MN là đường trung bình của tam giác ABC$\Rightarrow MN||AB\Rightarrow MK\perp EF$ (2)

Từ (1) và (2) suy ra E là trực tâm của tam giác MFK $\Rightarrow EM\perp FK\Rightarrow FK||BM$

Suy ra FKMB là hình bình hành $\Rightarrow đpcm$




#722862 BDT

Posted by toanND on 08-06-2019 - 16:49 in Bất đẳng thức và cực trị

mình nghĩ là có đk a,b,c không âm




#722984 Chứng minh rằng $\widehat{APB}=\widehat{APD...

Posted by toanND on 12-06-2019 - 10:40 in Hình học

$\boxed{\text{Bài toán}}$ [Bulgaria TST 2003] Cho tứ giác ABCD ngoại tiếp đường tròn (O). Hạ OP vuông góc AC. Chứng minh rằng $\widehat{APB}=\widehat{APD}$

Hình gửi kèm

Bulgaria TST 2003.PNG




#721264 $\sum \frac{a+b}{c} \geq 2\sqrt{( \sum a)(...

Posted by toanND on 03-04-2019 - 23:52 in Bất đẳng thức - Cực trị

Cho các số thực dương a,b,c. Chứng minh rằng khi đó ta có:

$\frac{a+b}{c}+\frac{b+c}{a}+\frac{c+a}{b}\geqslant 2\sqrt{(a+b+c)(\frac{a}{bc}+\frac{b}{ca}+\frac{c}{ab})}$




#724419 $1+\sqrt{\frac{2+1}{2}}+\sq...

Posted by toanND on 01-08-2019 - 21:05 in Bất đẳng thức và cực trị

 chỗ $\left ( 1+\frac{1}{n^{2}} \right )^{n}> 1+n.\frac{1}{n^{2}}$ này là sao bạn

Bất đẳng thức Bernoulli đó